Last visit was: 24 Apr 2024, 18:48 It is currently 24 Apr 2024, 18:48

Close
GMAT Club Daily Prep
Thank you for using the timer - this advanced tool can estimate your performance and suggest more practice questions. We have subscribed you to Daily Prep Questions via email.

Customized
for You

we will pick new questions that match your level based on your Timer History

Track
Your Progress

every week, we’ll send you an estimated GMAT score based on your performance

Practice
Pays

we will pick new questions that match your level based on your Timer History
Not interested in getting valuable practice questions and articles delivered to your email? No problem, unsubscribe here.
Close
Request Expert Reply
Confirm Cancel
SORT BY:
Date
Tags:
Difficulty: Sub 505 Levelx   Weakenx                           
Show Tags
Hide Tags
User avatar
Intern
Intern
Joined: 21 Dec 2006
Posts: 48
Own Kudos [?]: 226 [56]
Given Kudos: 0
Send PM
Most Helpful Reply
GMAT Club Verbal Expert
Joined: 13 Aug 2009
Status: GMAT/GRE/LSAT tutors
Posts: 6920
Own Kudos [?]: 63658 [11]
Given Kudos: 1773
Location: United States (CO)
GMAT 1: 780 Q51 V46
GMAT 2: 800 Q51 V51
GRE 1: Q170 V170

GRE 2: Q170 V170
Send PM
General Discussion
User avatar
Manager
Manager
Joined: 27 Jul 2006
Posts: 159
Own Kudos [?]: 36 [0]
Given Kudos: 0
Send PM
User avatar
Intern
Intern
Joined: 29 Jan 2007
Posts: 19
Own Kudos [?]: 15 [1]
Given Kudos: 0
Send PM
Re: An eyeglass manufacturer tried to boost sales for the summer [#permalink]
1
Kudos
E it is .
If the distributors are overstocked with the summers buying , they will not go for the discounted sale this time.

Acc. to me , B doesnt say that the reason for the success of the offer in summers was due to adverstising .
In general the reason of success is attributed to adverstising which can be possible in winters dicount plan as well.
~sara
User avatar
Intern
Intern
Joined: 11 Mar 2007
Posts: 28
Own Kudos [?]: 7 [0]
Given Kudos: 0
Send PM
Re: An eyeglass manufacturer tried to boost sales for the summer [#permalink]
At first i narrowed it down to A vs E & picked A. But then reading it more closely, i thought E is rite one.

"The distributors’ ordering more goods in the summer quarter left them overstocked for the fall quarter"

It clearly states that the distributor ordered in excess in Summber & now since he already has an inventory, he would not like to order more.
User avatar
Manager
Manager
Joined: 27 Mar 2007
Posts: 188
Own Kudos [?]: 157 [3]
Given Kudos: 0
Send PM
Re: An eyeglass manufacturer tried to boost sales for the summer [#permalink]
2
Kudos
1
Bookmarks
(A) Compares the summer quarter with the spring quarter. Out of scope, it is about the next fall quarter.
(B) The manufacturer could increase the newspaper and
radio advertising in those distributors’ sales areas during the fall quarter, this is no flaw.

(C) Irrelevant, as long as the sell enoough sunglasses
(D) Irrelevant, the manufacturer only wants the sunglasses to be sold at a certain discount
(E) Which could be the reason that the sunglasses don't sell. Either the costumers have already enough sunglasses at home, so that they won't buy more or the shops are overstocked, because they wanted to qualify for the discount. Best answer! Cheers
avatar
Director
Director
Joined: 29 Nov 2012
Posts: 580
Own Kudos [?]: 6041 [0]
Given Kudos: 543
Send PM
Re: An eyeglass manufacturer tried to boost sales for the summer [#permalink]
So the problem here is eyeglass manufactures plans to use same strategy as they did in the last quarter. What if the distributor over stocks supply, since last quarter there was a discount. The same strategy won't work this quarter.

Option E explains the Flaw perfectly :)
User avatar
Senior Manager
Senior Manager
Joined: 17 Apr 2013
Status:Verbal Forum Moderator
Posts: 361
Own Kudos [?]: 2197 [4]
Given Kudos: 298
Location: India
GMAT 1: 710 Q50 V36
GMAT 2: 750 Q51 V41
GMAT 3: 790 Q51 V49
GPA: 3.3
Send PM
Re: An eyeglass manufacturer tried to boost sales for the summer [#permalink]
4
Kudos
The secret in solving Critical reasoning questions lies in identifying correct conclusion, if you inculcate such habit you will sail more confidently with no ambiguity at all.
avatar
Intern
Intern
Joined: 17 Oct 2013
Posts: 6
Own Kudos [?]: 2 [0]
Given Kudos: 1
Send PM
Re: An eyeglass manufacturer tried to boost sales for the summer [#permalink]
Although I got my answer correct, but I pre-assumed something different than given in the answer choice. Please check whether this my pre-assumption is also correct:

The consumers had a demand for eyeglasses such as goggles in summer, but that demand reduced in fall as that was not the season goggles are meant for.

Thanks and Regards
Tushar
avatar
Intern
Intern
Joined: 15 Nov 2015
Posts: 30
Own Kudos [?]: 46 [0]
Given Kudos: 1
Send PM
Re: An eyeglass manufacturer tried to boost sales for the summer [#permalink]
manufacture concludes that fall quarter will have the same results as summer quarter.
Assumption: Everything will remain the same in fall as in summer.

Something along the lines of - in fall something is different compared to summer will undermine the assumption.
Senior Manager
Senior Manager
Joined: 15 Oct 2015
Posts: 375
Own Kudos [?]: 1551 [0]
Given Kudos: 342
Concentration: Finance, Strategy
GPA: 3.93
WE:Account Management (Education)
Send PM
Re: An eyeglass manufacturer tried to boost sales for the summer [#permalink]
B is a strengthener.
The ads will have affected final consumers buying from the resellers. Thus the stores sold well and will likely go for the blessed discount come fall.

B did not offer alternative cause as someone suggested.
Even if it did?
That's not what you are asked here.
But it didn't.

E said that the stores may not buy come fall cos they still got too many to sell.
The company's profit depends on the stores buying from them.

Posted from my mobile device
Retired Moderator
Joined: 22 Jun 2014
Posts: 971
Own Kudos [?]: 3801 [0]
Given Kudos: 182
Location: India
Concentration: General Management, Technology
GMAT 1: 540 Q45 V20
GPA: 2.49
WE:Information Technology (Computer Software)
Send PM
Re: An eyeglass manufacturer tried to boost sales for the summer [#permalink]
soxy_topacio wrote:
Official Guide for GMAT Verbal Review, 2nd Edition

Practice Question
Question No.: 83
Page: 151
Difficulty:


An eyeglass manufacturer tried to boost sales for the summer quarter by offering its distributors a special discount if their orders for that quarter exceeded those for last year's summer quarter by at least 20 percent. Many distributors qualified for this discount. Even with much merchandise discounted, sales increased enough to produce a healthy gain in net profits. The manufacturer plans to repeat this success by offering the same sort of discount for the fall quarter.

Which of the following, if true, most clearly points to a flaw in the manufacturer's plan to repeat the successful performance of the summer quarter?

(A) In general, a distributor's orders for the summer quarter are no higher than those for the spring quarter.
(B) Along with offering special discounts to qualifying distributors, the manufacturer increased newspaper and radio advertising in those distributors' sales areas.
(C) The distributors most likely to qualify for the manufacturer's special discount are those whose orders were unusually low a year earlier.
(D) The distributors how qualified for the manufacturer's special discount were free to decide how much of that discount to pass on to their own customers.
(E) The distributors' ordering more goods in the summer quarter left them overstocked for the fall quarter.


This question is based on the flaw in which author assumes what has been true in the past would be true in the future as well. This is a universal flaw unless a premise is added in the argument saying that the conditions of the past and present are exactly similar. With this thought in mind we should only be searching for the choice which tells us that present scenario is different from past because of some reason. Also to speed up, read choice A and E first (this is personal approach). In this question option E tells how present scenario is different from past. so it is correct.
Director
Director
Joined: 09 Jun 2010
Posts: 530
Own Kudos [?]: 523 [0]
Given Kudos: 916
Send PM
Re: An eyeglass manufacturer tried to boost sales for the summer [#permalink]
in a "plan" argument , author plan to do something, our job is to find something which support or weaken the plan. this job is more simple than criticize an argument in which evidence and a conclusion are offered.

in short, this kind of argument is more easy, normally.

the most difficult job and the main job we have to do is to criticize the argument . if we focus on the criticization we can solve many types of questions, assumption question, weaken/strengthen questions, flaw question and evaluate questions of of which are based on criticization.

plan argument is more easily, normally.
User avatar
Senior Manager
Senior Manager
Joined: 31 Mar 2016
Posts: 325
Own Kudos [?]: 195 [0]
Given Kudos: 197
Location: India
Concentration: Operations, Finance
GMAT 1: 670 Q48 V34
GPA: 3.8
WE:Operations (Commercial Banking)
Send PM
Re: An eyeglass manufacturer tried to boost sales for the summer [#permalink]
An eyeglass manufacturer tried to boost sales for the summer quarter by offering its distributors a special discount if their orders for that quarter exceeded those for last year's summer quarter by at least 20 percent. Many distributors qualified for this discount. Even with much merchandise discounted, sales increased enough to produce a healthy gain in net profits. The manufacturer plans to repeat this success by offering the same sort of discount for the fall quarter.
.
In weaken questions, usually there would be some sort of logical flaw like an undue or broad assumption that can be guessed. From the above highlighted portion, it is clear that the assumption based on which the manufacturer concluded to repeat the same strategy is that all the supplies to the distributors have been sold out so more number can be sold during the fall. Option E clearly attacks this conclusion by bringing in an alternative explanation that if distributors had over ordered the products due to discounts, then some or more of them would be still remaining with them even after sales necessary for profits. So if that turns out to be true, then that would weaken the conclusion the most.

Option E is the best choice.
Manager
Manager
Joined: 24 May 2016
Posts: 122
Own Kudos [?]: 421 [0]
Given Kudos: 33
Send PM
Re: An eyeglass manufacturer tried to boost sales for the summer [#permalink]
Options B and E are good candidates.

B) Along with offering special discounts to qualifying distributors, the manufacturer increased newspaper and radio advertising in those distributors' sales areas.
If the manufacturer's plan depends on this, the manufacturer could invest in a newspaper and radio advertising campaign during the fall season in order to mitigate this risk and push the distributors' sales. On such case, this would not be a flaw of the manufacturer's plan.
E) The distributors' ordering more goods in the summer quarter left them overstocked for the fall quarter.
Intern
Intern
Joined: 24 May 2014
Posts: 11
Own Kudos [?]: 7 [0]
Given Kudos: 378
Location: India
Concentration: General Management, Operations
WE:Engineering (Energy and Utilities)
Send PM
Re: An eyeglass manufacturer tried to boost sales for the summer [#permalink]
E it is.... clearly indicates that during next quarter the demand will be not more. so the plan of the manufacturer will not work.
Manager
Manager
Joined: 03 Jan 2017
Posts: 88
Own Kudos [?]: 83 [0]
Given Kudos: 4
Send PM
Re: An eyeglass manufacturer tried to boost sales for the summer [#permalink]
We need to find an answer giving reason why summer discount system will not work in fall
E-suggests one of the reasons
Manager
Manager
Joined: 26 Mar 2017
Posts: 62
Own Kudos [?]: 221 [0]
Given Kudos: 1
Send PM
Re: An eyeglass manufacturer tried to boost sales for the summer [#permalink]
catgmat wrote:
(A) Compares the summer quarter with the spring quarter. Out of scope, it is about the next fall quarter.
(B) The manufacturer could increase the newspaper and
radio advertising in those distributors’ sales areas during the fall quarter, this is no flaw.

(C) Irrelevant, as long as the sell enoough sunglasses
(D) Irrelevant, the manufacturer only wants the sunglasses to be sold at a certain discount
(E) Which could be the reason that the sunglasses don't sell. Either the costumers have already enough sunglasses at home, so that they won't buy more or the shops are overstocked, because they wanted to qualify for the discount. Best answer! Cheers


well, you can also argue that even if some distributors are overstocked its still possible that others will make use of the discount this time. They should make it clear. There are also enough arguments for B. Of course you can argue that it is a flaw. Maybe exactly these expenses were responsible for the sales increase. Its stated that the manufacturer plans to repeat this success by offering the same sort of discount for the fall quarter... So again they don't take into account that it might was the marketing expenses which were responsible for the increase in sales. Again niche clear to me.... gets my back up this stuff
Manager
Manager
Joined: 04 Oct 2015
Posts: 172
Own Kudos [?]: 148 [0]
Given Kudos: 242
Location: Viet Nam
Concentration: Finance, Economics
GMAT 1: 730 Q51 V36
GPA: 3.56
Send PM
Re: An eyeglass manufacturer tried to boost sales for the summer [#permalink]
An eyeglass manufacturer tried to boost sales for the summer quarter by offering its distributors a special discount if their orders for that quarter exceeded those for last year's summer quarter by at least 20 percent. Many distributors qualified for this discount. Even with much merchandise discounted, sales increased enough to produce a healthy gain in net profits. The manufacturer plans to repeat this success by offering the same sort of discount for the fall quarter.

Which of the following, if true, most clearly points to a flaw in the manufacturer's plan to repeat the successful performance of the summer quarter?

(A) In general, a distributor's orders for the summer quarter are no higher than those for the spring quarter.
--> irrelevant.

(B) Along with offering special discounts to qualifying distributors, the manufacturer increased newspaper and radio advertising in those distributors' sales areas.
--> this option seems to strengthen the argument.

(C) The distributors most likely to qualify for the manufacturer's special discount are those whose orders were unusually low a year earlier.

(D) The distributors how qualified for the manufacturer's special discount were free to decide how much of that discount to pass on to their own customers.
--> irrelevant.

(E) The distributors' ordering more goods in the summer quarter left them overstocked for the fall quarter.
--> correct.
Manager
Manager
Joined: 05 Jan 2016
Status:Final Call! Will Achieve Target ANyHow This Tym! :)
Posts: 70
Own Kudos [?]: 153 [0]
Given Kudos: 135
Location: India
GMAT 1: 620 Q49 V25
GPA: 3.8
Send PM
Re: An eyeglass manufacturer tried to boost sales for the summer [#permalink]
An eyeglass manufacturer tried to boost sales for the summer quarter by offering its distributors a special discount if their orders for that quarter exceeded those for last year's summer quarter by at least 20 percent. Many distributors qualified for this discount. Even with much merchandise discounted, sales increased enough to produce a healthy gain in net profits. The manufacturer plans to repeat this success by offering the same sort of discount for the fall quarter.

Which of the following, if true, most clearly points to a flaw in the manufacturer's plan to repeat the successful performance of the summer quarter?

FLAW Questions : Key Point -
Suppose X will lead to Y.
To point a flaw, you need to prove - X may / will not lead to Y.


Now, with respect to this question,
Offering discounts for summer quarters leads to increased sales thus increase in net profits.
Our task in hand for this question -- Offering discounts for fall quarter may / will not lead to increased sales thus increase in net profits. Now, look at the options keeping this approach in mind.

(A) In general, a distributor's orders for the summer quarter are no higher than those for the spring quarter.
This option says - No. of distributor's orders for the summer quarter are either less than or equal to the distributor's orders for the spring quarter. But are we concerned about this comparison? :?: :?:

For summer quarter, a special discount would be offered to manufacturer's distributors if their orders for that quarter exceeded those for last year's summer quarter by at least 20 percent. So, we are concerned about such a comparison and that too for the Summer quarter.

But Our conclusion is for the Fall quarter and we are not specifically given in the argument what exact special discount would be offered and what exactly would be the proportion of orders compared to last fall quarter. So, such a comparison is irrelevant. (the argument mentions only this -- the same sort of discount for the fall quarter.)

(B) Along with offering special discounts to qualifying distributors, the manufacturer increased newspaper and radio advertising in those distributors' sales areas.
What else the manufacturers did to boost sales for the summer quarter is totally irrelevant to prove a flaw in reasoning for the fall quarter. So, incorrect.

(C) The distributors most likely to qualify for the manufacturer's special discount are those whose orders were unusually low a year earlier.
This is talking about the distributors who qualified for the special discount. Again are we concerned for those distributors who qualified OR not qualified?? Does knowing this information will help in anyway to prove a flaw in manufacturer's reasoning for the fall quarter?? :?:

The Answer for the above question is YES. We are concerned who qualified and we need to be sure whether they will qualify this FALL Quarter or not.. for increased sales.. But how will this help in proving a flaw?? :idea: We need more details. So, Incorrect.

(D) The distributors who qualified for the manufacturer's special discount were free to decide how much of that discount to pass on to their own customers.
Totally irrelevant to the context in hand.

(E) The distributors' ordering more goods in the summer quarter left them overstocked for the fall quarter.
This proves that in the fall quarter, there will not be enough distributors' ordering goods in the fall quarter as they are overstocked and most of the distributors' who qualified for the special discount will be in the similar condition. This proves that -- Offering discounts for fall quarter may / will not lead to increased sales thus increase in net profits. Hence, this strengthens the mind set with which started looking into the options. Thus, this is the CORRECT ANSWER.
GMAT Club Bot
Re: An eyeglass manufacturer tried to boost sales for the summer [#permalink]
 1   2   
Moderators:
GMAT Club Verbal Expert
6920 posts
GMAT Club Verbal Expert
238 posts
CR Forum Moderator
832 posts

Powered by phpBB © phpBB Group | Emoji artwork provided by EmojiOne